If $\Phi\geq 0$ Is Non-decreasing, Does $\int_1^\infty \frac{\Phi(x)}{x ...

    1. Home
    2. Questions
    3. Tags
    4. Users
    5. Unanswered
  1. Teams

    Ask questions, find answers and collaborate at work with Stack Overflow for Teams.

    Try Teams for free Explore Teams
  2. Teams
  3. Ask questions, find answers and collaborate at work with Stack Overflow for Teams. Explore Teams

Teams

Q&A for work

Connect and share knowledge within a single location that is structured and easy to search.

Learn more about Teams If $\Phi\geq 0$ is non-decreasing, does $\int_1^\infty \frac{\Phi(x)}{x^2}\,dx=\infty$ imply $\int_0^\infty e^{-\Phi(x)}\,dx<\infty$? Ask Question Asked 3 years, 4 months ago Modified 3 years, 4 months ago Viewed 634 times 24 $\begingroup$

Q. Suppose $\Phi : [0, \infty) \to [0, \infty)$ is a non-decreasing function. If $$ \int_{1}^{\infty} \frac{\Phi(x)}{x^2} \, \mathrm{d}x = \infty, $$ then does the inequality $$ \int_{0}^{\infty} e^{-\Phi(x)} \, \mathrm{d}x < \infty $$ hold?

This question is motivated by this posting, but it became a statement of independent interest to me.

I suspect that the statement is true, based on a naive observation that a higher growth rate of $\Phi$ is in favor of both conditions and a slower growth rate is against them.

One obvious observation is that $\Phi(x)$ must tend to $\infty$ as $x \to \infty$. But, to be honest, I have no clue as to where I should start. So I would like to invite you to this strange yet interesting problem!

Edit. The user @Sungjin Kim showed that the answer is false by providing a family of counter-examples. Here is a simplification of his answer:

For each strictly increasing sequence $(b_k)_{k=1}^{\infty}$ in $[1, \infty)$, let

$$ \Phi(t) = \sum_{k \mathop{:} b_k \leq t} b_k = \sum_{k=1}^{\infty} b_k \mathbf{1}_{[b_k, \infty)}(t). $$

Then it follows that

$$ \int_{1}^{\infty} \frac{\Phi(t)}{t^2} \, \mathrm{d}t = \sum_{k=1}^{\infty} b_k \int_{b_k}^{\infty} \frac{\mathrm{d}t}{t^2} = \sum_{k=1}^{\infty} 1 = \infty. $$

On the other hand, since $\Phi(t) = \sum_{l=1}^{k} b_l$ for each $t \in [b_k, b_{k+1})$,

\begin{align*} \int_{0}^{\infty} e^{-\Phi(t)} \, \mathrm{d}t \geq \sum_{k=1}^{\infty} \int_{b_k}^{b_{k+1}} e^{-\Phi(t)} \, \mathrm{d}t = \sum_{k=1}^{\infty} (b_{k+1} - b_k) e^{-\sum_{l=1}^{k} b_l}. \end{align*}

So by choosing $(b_{k+1})$ so that it satisfies $(b_{k+1} - b_k) e^{-\sum_{l=1}^{k} b_l} \geq 1$ for each $k$, we have

$$ \int_{0}^{\infty} e^{-\Phi(t)} \, \mathrm{d}t = \infty. $$

Share Cite Follow edited Jul 25, 2021 at 15:54 Sangchul Lee asked Jul 22, 2021 at 20:06 Sangchul Lee's user avatar Sangchul LeeSangchul Lee 174k17 gold badges286 silver badges460 bronze badges $\endgroup$ 4
  • $\begingroup$ As an observation, there seems to be a 'breakeven' point where the first and second integrals are both convergent. This happens for functions like $\Phi(x)=2\ln(x+1)$. Maybe someone can use this to get some bounds on what $\Phi(x)$ can actually be $\endgroup$ – QC_QAOA Commented Jul 22, 2021 at 21:25
  • 1 $\begingroup$ Nice simplification! One typo is in $\Phi(t)=\sum_l b_k$. It should be $\sum_l b_l$. $\endgroup$ – Sungjin Kim Commented Jul 25, 2021 at 15:53
  • 1 $\begingroup$ But what if we require $\Phi$ to be smooth? Can this counterexample be modified to smooth out the jumps? $\endgroup$ – Simon Parker Commented Jul 27, 2021 at 19:07
  • 1 $\begingroup$ @SimonParker, We may mollify $\Phi$ to be smooth. For example, we may mollify $\Phi$ to a smooth function $\tilde{\Phi}$ such that $\Phi(x) \leq \tilde{\Phi}(x) \leq \Phi(x-1)$ and use $\tilde{\Phi}$ instead. A fairly simple comparison will then allow us to directly use our observation for $\Phi$. $\endgroup$ – Sangchul Lee Commented Jul 27, 2021 at 19:13
Add a comment |

1 Answer 1

Sorted by: Reset to default Highest score (default) Date modified (newest first) Date created (oldest first) 12 $\begingroup$

We may assume $\Phi(1)=0$. Let $\Phi(x)=\int_1^x f(t) dt$ for some nonnegative function $f$. By assumption, we have $$ \int_1^{\infty} \int_1^x \frac{f(t)}{x^2}dtdx =\int_1^{\infty}\int_t^{\infty} \frac{f(t)}{x^2} dxdt = \int_1^{\infty} \frac{f(t)}tdt=\infty. $$ Let $$ a_{\ell} = \int_{2^{\ell-1}}^{2^{\ell}} \frac{f(t)}t dt. $$ We have $\sum_{\ell=1}^{\infty} a_{\ell} = \infty$ and $$ \int_{2^{\ell-1}}^{2^{\ell}} f(t) dt \leq 2^{\ell}\int_{2^{\ell-1}}^{2^{\ell}} \frac{f(t)}t dt=2^{\ell}a_{\ell}. $$ On the other hand, by Cauchy condensation test, the convergence of $\int_1^{\infty} e^{-\Phi(x)} dx$ is equivalent to the convergence of LHS of $$ \sum_{k=1}^{\infty} 2^k e^{-\Phi(2^k)} \geq \sum_{k=1}^{\infty} 2^k e^{-\sum_{\ell\leq k} 2^{\ell}a_{\ell} }. $$ Let $a_{\ell}$ be a characteristic function of a very rapidly increasing sequence $\{b_n\}$ of natural numbers.

If $b_m\leq k < b_{m+1}$, we have $$ 2^k e^{-\sum_{\ell\leq k} 2^{\ell}a_{\ell} }\geq 2^{b_m} e^{-2^{1+b_m}}. $$ Then the sum over $k$ in this interval is $$ \geq (b_{m+1}-b_m)2^{b_m} e^{-2^{1+b_m}}. $$ For each $m$, we take $b_{m+1}$ large enough that $$ (b_{m+1}-b_m)2^{b_m} e^{-2^{1+b_m}}\geq 1. $$ Then we have the divergence of the series $$ \sum_{k=1}^{\infty} 2^k e^{-\sum_{\ell\leq k} 2^{\ell}a_{\ell} }. $$ Hence, we have the divergence of $$ \int_1^{\infty} e^{-\Phi(x)} dx. $$

Share Cite Follow answered Jul 25, 2021 at 14:33 Sungjin Kim's user avatar Sungjin KimSungjin Kim 20.6k3 gold badges29 silver badges73 bronze badges $\endgroup$ 4
  • 1 $\begingroup$ I am not claiming the divergence in all case. I showed a counterexample. $\endgroup$ – Sungjin Kim Commented Jul 25, 2021 at 14:52
  • 1 $\begingroup$ Great answer! It is interesting to have a counter-example. :) $\endgroup$ – Sangchul Lee Commented Jul 25, 2021 at 15:03
  • $\begingroup$ I tried to simplify your answer by myself and updated my posting. Again, thank you for your elegant idea! $\endgroup$ – Sangchul Lee Commented Jul 25, 2021 at 15:09
  • $\begingroup$ @SungjinKim Ahh sry, I misunderstood what you were doing. $\endgroup$ – Maximilian Janisch Commented Jul 25, 2021 at 16:13
Add a comment |

You must log in to answer this question.

Not the answer you're looking for? Browse other questions tagged .

  • Featured on Meta
  • We’re (finally!) going to the cloud!
  • More network sites to see advertising test [updated with phase 2]

Linked

10 Is $\int_{\mathbb{R}^2} e^{-u} \Delta u < \infty$? 3 On the absolute integrability of radially symmetric functions 1 Constructing a specific normalised test function. 5 On the integral $\int_{0}^{\pi/2} \frac{x \log \left ( 1-\sin x \right )}{\sin x} \, \mathrm{d}x$ 2 Prove $\sum_{n\le x} d(n)/n = \frac12 \log^2 x + 2e \log x + c + \mathcal O(1/\sqrt{x})$ where $d$ is the divisor function and $c$ is a constant 3 A counterexample on tails of distribution 2 Limits using layer-cake representations for $L^p$ space involving two functions 4 About an integral from the MIT integration bee 2022 1 Integrating $\int_{0}^{\infty} \mathrm{d}x \frac{x^{2 k +1}}{\cosh \left( x- y\right)} $

Hot Network Questions

  • Can Windows firewall WF.msc do TCP port forwarding at all, like Linux iptables does?
  • Is ATL-98 Carvair still alive in the US?
  • What happened to the lifeboats in Star Trek: First Contact?
  • Simultaneously cooling and humidifying in winter?
  • How is one supposed to play these notes?
  • Non-reflexive use of laisser without a direct object in « The Stranger » ?
  • What is the proper way to say "voice direction" in German?
  • RegionIntersection of 3D objects
  • Custom document class always pick the default logo even if I change to other logos
  • Full Bridge Rectifier - Output Voltage Saturation
  • Difference between English short stories and short English stories
  • Spacesuit for Vermoid Locomotion
  • Implicit function theorem without manifolds (Steve Smale article)?
  • Use of “12 m.” for noon and “12 p.m.” for midnight
  • Formative alternative to midterms for a large class
  • will "brown aluminum" drip-edge suffer galvanic corrosion if it rests against fascia made of copper-treated lumber?
  • How to identify unsafe trees for climbing stand?
  • Can .zshrc be modified automatically by other programs, installers, etc.?
  • Why is this soldering iron red hot in the middle section?
  • Using Reverse Tunnel for accessing URL not directly accessible
  • Can basic authentication somehow be made to remember the login when closing the browser?
  • What's the contrary of formation as in the opposite of the process of formation?
  • Why doesn't gas usually contract in volume?
  • Baskervaldx doesn’t allow certain glyphs
more hot questions Question feed Subscribe to RSS Question feed

To subscribe to this RSS feed, copy and paste this URL into your RSS reader.

Từ khóa » Dx Phi